caduta e oscillazione

Meccanica, termodinamica, elettromagnetismo, relatività, ...
Rispondi
Riccardo_ct
Messaggi: 46
Iscritto il: 23 nov 2007, 15:04

caduta e oscillazione

Messaggio da Riccardo_ct »

Un corpo di massa $ m $ cade da un'altezza $ h $ su una molla di costante elastica $ k $. Arrivata sulla sommità della molla la massa si attacca ad essa (l'urto quindi è anelastico) e comincia ad oscillare nella direzione verticale.
Trovare l'ampiezza e l'energia di queste oscillazioni.


----------------------------------------------------------
S'impara molto dai suggerimenti, ma ancora di più s'impara dai propri errori: ho cancellato i risultati per dare agli utenti il tempo di cimentarsi nel problema.
Mathomico
Rigel
Messaggi: 74
Iscritto il: 19 mag 2008, 17:33
Località: Sternatia

Messaggio da Rigel »

Scusa, ma la massa della molla è trascurabile?
Se così fosse le velocità del corpo prima e dopo l'urto sarebbero uguali...
"Non ho particolari talenti, sono solo appassionatamente curioso." Albert Einstein
Riccardo_ct
Messaggi: 46
Iscritto il: 23 nov 2007, 15:04

Messaggio da Riccardo_ct »

si scusa mi sono dimenticato di specificarlo, la massa della molla è trascurabile...
Rigel
Messaggi: 74
Iscritto il: 19 mag 2008, 17:33
Località: Sternatia

Messaggio da Rigel »

Visto che nessuno ha ancora scritto una soluzione...
Poniamo l'energia potenziale $ $U$ $ uguale a 0 alla sommità della molla a riposo.
Sia $ $K$ $ l'energia cinetica e $ $E$ $ quella elastica della molla.
L'urto tra corpo e molla e anelastico, ma la massa dlla molla è trascurabile e quindi la velocità del corpo non cambia.
Pertanto quando urta la molla, il corpo ha energia cinetica $ $K=mgh$ $ e velocità $ $v=\sqrt{2gh}$ $.
Per la conservazione dell'energia, l'energia totale delle oscillazioni non cambia nel tempo (trascurando eventuali attriti) ed è data dalla somma di $ $U, K, E$ $ in qualsiasi momento del moto. Considerando il momento in cui il corpo urta la molla si ha $ $E_{T}=U_{0}+K_{0}+E_{0}=0+mgh+0=mgh$ $.
Per quanto riguarda l'ampiezza $ $A$ $, essa è data dalla compressione $ $\Delta x$ $ della molla quando la velocità (e quindi $ $K$ $) è nulla. sempre per la conservazione dell'energia $ $U_{1}+K_{1}+E_{1}=U_{0}+K_{0}+E_{0}$ $, cioè
$ $U_{1}+E_{1}=K_{0}$ $
$ $-mg\Delta x+\frac{1}{2}k(\Delta x)^2=mgh$ $
$ $A=\Delta x=\frac{mg\pm\sqrt{m^2g^2+2kmgh}}{k}=\frac{mg+\sqrt{mg(mg+2kh)}}{k} $
Visto che$ $\sqrt{m^2g^2+2kmgh}>mg$ $ non considero la soluzione negativa.
"Non ho particolari talenti, sono solo appassionatamente curioso." Albert Einstein
Riccardo_ct
Messaggi: 46
Iscritto il: 23 nov 2007, 15:04

Messaggio da Riccardo_ct »

ero giunto anch'io alla stessa soluzione, ma purtroppo non coincide col risultato che mi fornisce il testo :( .
(proprio per questo motivo avevo postato i risultati)
Rigel
Messaggi: 74
Iscritto il: 19 mag 2008, 17:33
Località: Sternatia

Messaggio da Rigel »

Bè se sul tuo testo c'è solo una soluzione numerica è possibile che sia un errore di stampa o di calcolo del libro... con i libri migliori succede raramente ma è sempre una possibilità :wink:
Se invece è riportata un risultato con grandezze variabili (tipo quello che ho trovato io) sono proprio curioso di sapere la soluzione del testo... e anche la spiegazione.

A proposito che libro è?
"Non ho particolari talenti, sono solo appassionatamente curioso." Albert Einstein
Avatar utente
Mathomico
Messaggi: 216
Iscritto il: 01 gen 1970, 01:00
Località: Pisa

Messaggio da Mathomico »

Rigel ha scritto: Per quanto riguarda l'ampiezza $ $A$ $, essa è data dalla compressione $ $\Delta x$ $ della molla
Questo è vero, supponendo che la posizione in cui si trova la pallina quando sbatte contro la molla sia quella che risulterà di equilibrio per l'oscillazione.
Tuttavia così non è, e bisogna tener conto di questo fatto.

Mathomico
derfisc.
Messaggi: 14
Iscritto il: 30 apr 2007, 22:34

Messaggio da derfisc. »

mi esce $ A= \frac {mg} {k}\sqrt {1+\frac {2hk} {mg}} $ Ho lavorato con l' equazione differenziale $ mg-kx=mx'' $.
Ma forse non era richiesto questo.
Faust
Messaggi: 44
Iscritto il: 01 giu 2005, 19:50

Messaggio da Faust »

La butto là:
risultato di Rigel OK.

Tuttavia:

k(x^2)=mgA
Ich bin der geist, der stets ferneint!
Rigel
Messaggi: 74
Iscritto il: 19 mag 2008, 17:33
Località: Sternatia

Messaggio da Rigel »

Mathomico ha scritto:Questo è vero, supponendo che la posizione in cui si trova la pallina quando sbatte contro la molla sia quella che risulterà di equilibrio per l'oscillazione.
Tuttavia così non è, e bisogna tener conto di questo fatto.
Oooops :oops: Pardon errore mio e non del libro.
Vedrò di rimediare a questo sbagli grossolano...
L'ampiezza vera delle oscillazioni è data dall'ampiezza (sbagliata) che ho trovato prima meno la distanza tra sommità della molla e centro di equilibrio del moto armonico. Chiamiamo $ x $ tale distanza; il centro di equilibrio è il punto in cui la risultante delle forze agenti sul corpo è nulla, cioè $ $mg-kx=0\Rightarrow x=\frac{mg}{k}$ $. Adesso rettifico l'espressione dell'ampiezza
$ $A=\frac{mg+\sqrt{mg(mg+2kh)}}{k}-\frac{mg}{k}=\frac{\sqrt{mg(mg+2kh)}}{k}$ $
Questo risultato coincide con quello trovato da derfisc.
Spero di essere stato chiaro... anche se questa spiegazione non è tra le più scorrevoli :roll:
"Non ho particolari talenti, sono solo appassionatamente curioso." Albert Einstein
Avatar utente
Mathomico
Messaggi: 216
Iscritto il: 01 gen 1970, 01:00
Località: Pisa

Messaggio da Mathomico »

derfisc. ha scritto:mi esce $ A= \frac {mg} {k}\sqrt {1+\frac {2hk} {mg}} $
Concordo.
Rispondi